Jump to content

minhtuyb's Content

There have been 497 items by minhtuyb (Search limited from 07-06-2020)



Sort by                Order  

#298739 CMR:$cosA+cosB+ cos C \leq \frac{3}{2}$

Posted by minhtuyb on 09-02-2012 - 18:02 in Hình học

Bài này xuất hiện nhiều rồi, bạn search là thấy:

Attached File  VNMATH.COM-12cachchung minh bdt.pdf   69.26KB   81 downloads



#318985 Giải phương trình nghiệm nguyên dương $\frac{x}{y+21}+\frac{y}...

Posted by minhtuyb on 24-05-2012 - 11:21 in Số học

SOLUTION:
Đặt $x=a;y=b;21=c;6=d$, khi đó pt đã cho trở thành:
$$\frac{a}{b+c}+\frac{b}{c+d}+\frac{c}{a+d}+\frac{d}{a+b}=\frac{2}{z}(*)$$
Mặt khác, ta dễ dàng cm $VT(*)\ge 2$ theo Nesbit 4 biến và $VP(*)\le 2$ do $z\in N^*$. Dấu đẳng thức xảy ra khi và chỉ khi:
$$\left\{\begin{matrix}a=c\\ b=d\\ z=1\end{matrix}\right.\Leftrightarrow \left\{\begin{matrix}x=21\\ y=6\\ z=1\end{matrix}\right.$$
Vậy pt đã cho có nghiệm $x=21;y=6;z=1$



#416438 Chứng minh: $I,J,D$ thẳng hàng

Posted by minhtuyb on 04-05-2013 - 18:32 in Hình học

Cho đường tròn $(O)$. $A$ lad một điểm nằm ngoài đường tròn. Từ $A$ vẽ tiếp tuyến $AA'$ và cát tuyến $ACD$ ($C$ nằm giữa $A$ và $D$). Đường tròn $(A; AA')$ cắt đường thẳng $CD$ tại $E,F$ ($E$ nằm giữa  $C$ và $D$). Gọi $M=A'E\cap OF, N=DM\cap A'F, I=CM\cap A'F, J=NC\cap A'E$. Chứng minh: $I,J,D$ thẳng hàng.

Có hình luôn đây  :icon10:

- Hai đường tròn $(O)$ và $(A)$ trực giao nên $(FECD)=-1$

$\Rightarrow M(FECD)=-1\Rightarrow (FA'NI)=-1$

- Theo hệ quả của phép chiếu xuyên tâm thì $I,D,J$ thẳng hàng $\square$




#382542 $2\sum a\geq \sum \sqrt[3]{a^{3}+7...

Posted by minhtuyb on 01-01-2013 - 13:05 in Bất đẳng thức và cực trị

Cho $a,b,c,d>0$ thỏa
$a+b+c+d = \frac{1}{a^{2}}+\frac{1}{b^{2}}+\frac{1}{c^{2}}+\frac{1}{d^{2}}$
CMR
$2(a+b+c+d)\geq \sqrt[3]{a^{3}+7}+\sqrt[3]{b^{3}+7}+\sqrt[3]{c^{3}+7}+\sqrt[3]{d^{3}+7}$

Bài này sử dụng $Chebyshev$ là ra :):
---
-Viết lại giả thiết: $\sum \left(a-\dfrac{1}{a^2}\right)=0$. Ta có:
$$bdt\Leftrightarrow \sum (2a-\sqrt[3]{a^3+7})\ge 0\\ \Leftrightarrow \sum \dfrac{8a^3-(a^3+7)}{4a^2+2a\sqrt[3]{a^3+7}+\sqrt[3]{(a^3+7)^2}}\ge 0\\ \Leftrightarrow \sum \dfrac{a-\dfrac{1}{a^2}}{4+2\sqrt[3]{1+\dfrac{7}{a^3}}+\sqrt[3]{(1+\dfrac{7}{a})^2}}\ge 0$$
Giả sử $a\ge b\ge c\ge d$ thì ta có hai dãy đơn điệu cùng chiều:
$$a-\dfrac{1}{a^2}\ge b-\dfrac{1}{b^2}\ge c-\dfrac{1}{c^2}\ge d-\dfrac{1}{d^2}\
\\\dfrac{1}{4+2\sqrt[3]{1+\dfrac{7}{a^3}}+\sqrt[3]{(1+\dfrac{7}{a})^2}}\ge \dfrac{1}{4+2\sqrt[3]{1+\dfrac{7}{b^3}}+\sqrt[3]{(1+\dfrac{7}{b})^2}}\ge ...$$
Áp dụng BĐT $Chebyshev$ cho hai dãy đơn điệu cùng chiều ta có ĐPCM. Dấu bằng xảy ra khi $a=b=c=d=1$



#319510 chứng minh S không có quá 1 số lẻ

Posted by minhtuyb on 25-05-2012 - 20:16 in Các dạng toán khác

Giả sử $S$ có không ít hơn 2 số lẻ. Gọi 2 số lẻ đó là $a,b$. Vì tổng 2 phần tử bất kì trong $S$ là số chính phương nên có thể đặt:
$$a^2+b^2=k^2\ (k\in N^*)(1)$$
Xét modul 4:
$+)a^2\equiv 1\ (mod 4);b^2\equiv 1\ (mod 4)\ \text{Do a,b lẻ}\Rightarrow VT(1)\equiv 2\ (mod 4)(*)\\ +)k^2\equiv 0,1\ (mod 4) (**)$
-Vì $(*)$ và $(**)$ mâu thuẫn nên điều giả sử là sai. Vậy ta có ĐPCM



#296520 Tìm giá trị biểu thức: $P=\dfrac{1}{x}+\dfrac{1}{y}+\dfra...

Posted by minhtuyb on 26-01-2012 - 11:45 in Phương trình, hệ phương trình và bất phương trình

Cho x,y,z khác không thỏa mãn:
$\left\{\begin{matrix}
x(\dfrac{1}{y}+\dfrac{1}{z})+y(\dfrac{1}{x}+\dfrac{1}{z})z(\dfrac{1}{x}+\dfrac{1}{y})=-2 & \\
x^{2}+y^{2}+z^{2}=1 &
\end{matrix}\right.$
Tìm giá trị biểu thức: $P=\dfrac{1}{x}+\dfrac{1}{y}+\dfrac{1}{z}$

Tính đc đến đây thôi:
$x(\dfrac{1}{y}+\dfrac{1}{z})+y(\dfrac{1}{x}+\dfrac{1}{z})z(\dfrac{1}{x}+\dfrac{1}{y})=-2\Leftrightarrow \sum \frac{x}{y}+\frac{x}{z}=-2\Leftrightarrow \sum \frac{x}{x}+\frac{x}{y}+\frac{x}{z}=1$
$\Leftrightarrow \sum x(\frac{1}{x}+\frac{1}{y}+\frac{1}{z})=1\Leftrightarrow (x+y+z)(\frac{1}{x}+\frac{1}{y}+\frac{1}{z})=x^2+y^2+z^2(=1)
\Leftrightarrow \frac{1}{x}+\frac{1}{y}+\frac{1}{z}=\frac{x^2+y^2+z^2}{x+y+z}$



#300895 $CMR$ với mọi số nguyên $m$ thì :

Posted by minhtuyb on 25-02-2012 - 11:07 in Số học

$4m^3+9m^2-9m-30=(4m^3+3m^2-m)+(6m^2-18m-30)$
-Thấy $6m^2-18m-30=6(m^2-3m-5)\vdots 6\forall m\in Z$
-Giờ ta phải c/m: $4m^3+3m^2-m\vdots 6$. Thật vậy, có:
$4m^3+3m^2-m=4m^3+4m^2-m^-m=4m(m^2+m)-(m^2+m)=(4m-1)(m^2+m)=m(m+1)(4m-1)(1)$
-Vì $m(m+1)$ là tích 2 số nguyên liên tiếp nên chia hết cho 2
-Xét các trường hợp:
+)$m=3k\Rightarrow m\vdots 3$
+)$m=3k+1\Rightarrow 4m-1=4(3k+1)-1=12k+3\vdots 3$
+)$m=3k+2\Rightarrow m+1=3k+3\vdots 3$
Vậy trong mọi TH đều có $m(m+1)(4m-1)\vdots 3$
-Mà $(2;3)=1\Rightarrow m(m+1)(4m-1)\vdots 6$
Tóm lại bài toán đã đc c/m



#371912 Các bài toán về vecto

Posted by minhtuyb on 23-11-2012 - 21:17 in Hình học phẳng

bài1:Cho tam giác ABC có bán kính đường tròn ngoại tiếp là R,trọng tâm G.tìm vị trí điểm P để biểu thức http://latex.codecog...rac{3PG^2}{R^4}đạt giá trị bé nhất


bài 2:Cho tam giác ABC nội tiếp (O) ,I là tâm đường tròn nội tiếp tam giác.IA,IB,IC cắt (O) tại D,E,F.(khác A,B,C).Chứng minh rằng:
http://latex.codecog...9}{S\Delta ABC}

bài 3:Cho tam giác ABC,trọng tâm G,điểm lemoine L(giao của 3 đường đối trung).Chứng minh rằng:
http://latex.codecog...c{LC}{GC}\leq 3

Học gõ $\LaTeX$ và suy nghĩ kĩ trước khi hỏi bài!
Bài nào thử mọi cách ko ra thì mới hỏi. Đằng này BTVN chiều thầy vừa cho xong mà tối đã post lên đây rồi Posted Image.



#328034 Viết sau sô 1993 một số tự nhiên $a$ thì được số mới chia hết cho 1...

Posted by minhtuyb on 22-06-2012 - 18:37 in Số học

Viết sau sô 1993 một số tự nhiên $a$ thì được số mới chia hết cho 101:

a) Tìm GTNN của $a$
b) Nêu dấu hiệu chia hết cho $101$,chứng minh
c) Tìm công thức tổng quát của $a$

---------------
Thực ra bài thật thì chỉ có phần a), nhưng bạn nào có khả năng giúp mình phần $b);c)$ nữa nhé ^_^



#363713 Hệ thức Viet thi - HSG tỉnh Phú Thọ, chứng minh nghiệm này bằng k lần nghiệm...

Posted by minhtuyb on 21-10-2012 - 21:25 in Phương trình - hệ phương trình - bất phương trình

*Gợi ý: Gọi $x_1;x_2$ là 2 nghiệm của $(1)$, $x_3,x_4$ là 2 nghiệm của $(2)$.
+) Đ/k 2 pt có nghiệm: ...
+) Đ/k để $\alpha=k\beta:$
$$(x_1-kx_3)(x_2-kx_3)(x_1-kx_4)(x_2-kx_4)=0$$
Đến đây cần trâu bò là ra :D.



#356002 Từ các chữ số $0\rightarrow 6$ lập được bao nhiêu số tự nhiên...

Posted by minhtuyb on 22-09-2012 - 23:25 in Tổ hợp - Xác suất và thống kê - Số phức

- Gọi $\overline{a_1a_2a_3a_4a_5a_6a_7a_8a_9}$ là một số có $9$ chữ số thỏa mãn bài toán
- Số các số có $9$ chữ số trong đó có chữ số 1 có mặt đúng 3 lần, các chữ số còn lại có mặt đúng 1 lần, tính cả trường hợp $a_1=0$ là:
$$d_1=\overline{P}=\dfrac{9!}{3!}$$
- Số các số có $9$ chữ số trong đó có chữ số 1 có mặt đúng 3 lần, các chữ số còn lại có mặt đúng 1 lần, có $a_1=0$ là:
$$d_2=\overline{P}=\dfrac{8!}{3!}$$
Vậy có: $d=d_1-d_2=53760$ số thoả mãn yêu cầu bài toán $\square$



#300308 Tìm $x$ để $B$ = $0$

Posted by minhtuyb on 21-02-2012 - 14:03 in Số học

Bài này..... :wacko: ;
$$B=(x+3)(x^2+4x+4)=0\Leftrightarrow (x+3)(x+2)^2=0$$
Vậy với $x=-2;-3$ thì B bằng không
P/s:Đơn giản là phương trình tích thôi :wub:



#300753 Tìm $x$ để $B$ = $0$

Posted by minhtuyb on 24-02-2012 - 12:18 in Số học

minhtuyb giải hơi tắt . Cái $(x^2+4x+4)$ = $(x+2)^2$ thì mình biết rồi . Nhưng phải giải như theo trình tự mẫu này chứ cô giáo bảo mình học qua lớp 8 mới được giải như kiểu của bạn được :

Học qua lớp 8 là được làm tắt à, vậy mình giải đúng rồi còn j` :wub:
Chắc em học lớp 7, anh lớp 9 nên giải tắt tí cũng chả sao cả :icon6:
P/s:
$A(x)B(x)C(x)...Z(x)\neq 0\Leftrightarrow \left\{\begin{matrix}A(x)\neq 0\\B(x)\neq 0\\C(x)\neq 0 \\...\\Z(x)\neq 0\end{matrix}\right.$



#358450 Chứng minh với mọi số tự nhiên $n$ lẻ: $2^{n!}-1...

Posted by minhtuyb on 02-10-2012 - 23:08 in Số học

Thử mở rộng bài toán với:

"Tìm mọi số tự nhiên $n$ sao cho:
$$2^n-1\vdots n$$"



#358456 Chứng minh với mọi số tự nhiên $n$ lẻ: $2^{n!}-1...

Posted by minhtuyb on 02-10-2012 - 23:25 in Số học

Giải như sau:
Bổ đề quen thuộc: $a^x-1,a^y-1 \vdots p$ với $x$ min thì $x|y$
Gọi $p$ là ước nguyên tố của $n$, dễ cm $n$ lẻ suy ra $p$ lẻ và $p$ là ước nguyên tố bé nhất của $n$
Khi đó gọi $k$ là số nhỏ nhất thỏa mãn $2^k-1 \vdots p$ theo Fermat nhỏ $2^{p-1}-1 \vdots p$
Nên theo bổ đề quen thuộc có $p-1 \vdots k \Rightarrow p-1\geq k \Rightarrow p>k$
Mặt khác cũng theo bổ đề đó có $n \vdots k$
Suy ra $k=1$ vì nếu $k>1$ thì $k \vdots r$ nguyên tố lẻ mà $p>k\geq r \Rightarrow p>r$ mà $r|n$ vô lí vì $p$ là ước nguyên tố nhỏ nhất của $n$
Suy ra $2^k-1=2-1=1 \vdots p$ vô lí
Vậy vô nghiệm

Thanks em :D. Nhưng em xét thiếu trường hợp $n$ không có ước nguyên tố. Tức là $n=1$ :P



#298467 Cực trị hình học và hình chiếu của một điểm trên 3 cạnh tam giác nội tiếp

Posted by minhtuyb on 07-02-2012 - 13:15 in Hình học

Hóa ra hai bài này phải xài Sim-sơn :D.
Mình thắc mắc tí:

* $\frac{b}{y} +\frac{c}{z}=\frac{AC}{PL}+\frac{AB}{PM}=\frac{AL}{PL}+\frac{LC}{PL}+\frac{AM}{PM}-\frac{BM}{PM}$

Chỗ này chắc bạn hơi nhầm tí: $\frac{AC}{PL}+\frac{AB}{PM}=\frac{AL}{PL}+\frac{LC}{PL}+\frac{AM}{PM}+\frac{BM}{PM}$ (Nhầm dấu cộng thành trừ)

Dù sao cũng cảm ơn bạn nhiều :)

P/s:

Từ (1)(2)(3) $\frac{a}{x}=\frac{b}{y}+\frac{c}{z}\Rightarrow S=2.\frac{a}{x}$

Sao chỗ này mình thay góc vào không đc nhỉ. $\widehat{CPK}$ thay bằng góc gì đó



#298256 Cực trị hình học và hình chiếu của một điểm trên 3 cạnh tam giác nội tiếp

Posted by minhtuyb on 05-02-2012 - 22:24 in Hình học

Có 2 bài xin thình giáo :D
Bài 1:Cho $\Delta ABC$ vuông tại A nội tiếp đường tròn tâm O đường kính BC. P là một điểm di chuyển trên nửa đường tròn không chứa A. Gọi $K,L,M$ lần lượt là hình chiếu của P trên $BC,AC,AB$. Đặt $PK=x;PL=y;PM=z;BC=a;AC=b;AB=c$. Tìm GTNN của:
$S=\frac{a}{x}+\frac{b}{y}+\frac{c}{z}$
Bài 2: Cho Cho $\Delta ABC$ nội tiếp đường tròn tâm O. Điểm M nằm trên cung nhỏ AC. Kẻ $ME\perp AC;MF\perp BC$. Gọi $P,Q$ lần lượt là trung điểm của $AB,EF$.
CMR:$\Delta PQM$ vuông



#298571 chứng minh rằng trong các tổng của 5 số theo mỗi cột , mỗi hàng hay mỗi đường...

Posted by minhtuyb on 08-02-2012 - 13:17 in Các dạng toán khác

-Người ta viết vào mỗi ô vuông một trong các số -1,1,hoặc 0 nên có thể xảy ra trường hợp các tổng của 5 số có thể là: $ -5;-4;-3;-2;-1;0;1;2;3;4;5$ (11 trường hợp)
-Tổng cộng có: 5 tổng hàng+5 tổng cột+2 tổng đường chéo=12 tổng. +Theo diricle có ĐPCM



#303644 Chứng minh rằng $B = 3 + {3^3} + {3^5} + ... + {3^{1991}} \vdots 13...

Posted by minhtuyb on 11-03-2012 - 20:26 in Đại số

$B=(3+3^3+3^5)+(3^7+3^9+3^{11})+...+(3^{1987}+3^{1989}+3^{1991})$
$=3(1+3^2+3^4)+3^7(1+3^2+3^4)+...+3^{1987}(1+3^2+3^4)$
$=91(3+3^7+...+3^{1987})\vdots 13$ (Do $91\vdots 13$)



#321996 Tìm vị trí của điểm C trên cung nhỏ AB để CK.AD + CE.BD có GTLN.

Posted by minhtuyb on 03-06-2012 - 11:43 in Hình học

Cho đường tròn O dây AB cố định khác đường kính. Trên cung nhỏ AB lấy C, vẽ dây CD vuông góc với AB tại H, CK vuông góc với AD tại K. E là giao điểm của HK và BD.
Tìm vị trí của điểm C trên cung nhỏ AB để CK.AD + CE.BD có GTLN.

SOLUTION:
-Ta dễ dàng c/m $CE\perp AB$ (đây có thể nói là bài toán ngược của đường thẳng Sim-sơn)
Sau khi c/m xong ta có:
$$CK.AD+CE.BD=2S_{ACD}+2S_{BCD}=2S_{ACBD}=AB.CD$$
Mà $AB$ có độ dài không đổi (gt) nên $CK.AD+CE.BD$ đạt GTLN khi và chỉ khi $CD$ đạt GTLN
Mặt khác, $CD\le 2R$ với $R$ là bán kính đường tròn $(O)$ (theo quan hệ giữa dây và đường kính trong đường tròn)
Suy ra $CK.AD+CE.BD\le 2R.AB=const$
Dấu bằng xảy ra khi $CD$ là đường kính của đường tròn $(O)$
Vậy $max(CK.AD+CE.BD)=2R.AB$ khi $CD$ là đường kính của đường tròn $(O)$



#302023 tìm giá trị lớn nhất của $\sqrt{x+1}-\sqrt{x-8}$

Posted by minhtuyb on 03-03-2012 - 17:57 in Đại số

ĐKXĐ: $x\geq 8$
Nhân liên hợp, ta có:
$A=\sqrt{x+1}-\sqrt{x-8}=\frac{x+1-(x-8)}{\sqrt{x+1}+\sqrt{x-8}}=\frac{9}{\sqrt{x+1}+\sqrt{x-8}}$
Vì $x\geq 8\Rightarrow \sqrt{x+1}+\sqrt{x-8}\geq \sqrt{8+1}+\sqrt{8-8}=3\Rightarrow \frac{9}{\sqrt{x+1}+\sqrt{x-8}}\leq \frac{9}{3}=3$
Dấu bằng xảy ra khi $x=8$
Vậy $maxA=3$ khi $x=8$



#302895 tìm giá trị lớn nhất của $\sqrt{x+1}-\sqrt{x-8}$

Posted by minhtuyb on 08-03-2012 - 14:51 in Đại số

http://diendan.hocma...ad.php?t=184813
http://vn.answers.ya...29105656AAWtTaa
Tham khảo thêm nhé bạn :D



#371017 Chứng minh rằng có vô hạn các số có dạng $a_n=2^n-3$ ($n...

Posted by minhtuyb on 20-11-2012 - 19:46 in Số học

Chứng minh rằng có vô hạn các số có dạng $a_n=2^n-3$ ($n \geq 2$) đôi một nguyên tố cùng nhau

Xét : $n>1$ thì $a_n$ lẻ. Ta có hai giá trị khởi đầu:
$$a_2=1;a_3=5\Rightarrow (a_2;a_3)=1$$

Ta sẽ chứng minh: Giả sử ta chọn được dãy $b_1,b_2,...,b_k$ là dãy con của dãy $a_n$ thỏa mãn các phần tử của dãy đôi một nguyên tố cùng nhau. Ta sẽ chứng minh $\exists b_{k+1}$ đôi một nguyên tố cùng nhau với $b_1,b_2,...,b_k$. Thật vậy, đặt:
$$A=b_1.b_2....b_k\\ \Rightarrow (A,2)=1$$
Áp dụng định lí Euler:
$$2^{\varphi(A)}-1\vdots A$$
Mà $A$ lẻ $\Rightarrow (2^{\varphi(A)}-3,A)=1\Rightarrow 2^{\varphi(A)}-3$ đôi một nguyên tố cùng nhau với $b_1,b_2,...,b_k$.
Vậy ta chọn $b_{k+1}=\varphi(A)$.
Ta đã c/m được sự vô hạn của dãy $b_k$. Vậy mệnh đề được c/m $\square$



#321189 $\frac{1}{1+\sqrt{2}}...+\frac{1}{\sqrt{2n+1}+\...

Posted by minhtuyb on 31-05-2012 - 11:15 in Bất đẳng thức và cực trị

CMR $\frac{1}{1+\sqrt{2}}+\frac{1}{\sqrt{3}+\sqrt{4}}+...+\frac{1}{\sqrt{2n+1}+\sqrt{2n+2}}<\frac{ \sqrt{2n+2}}{2}$

Có: $\frac{2}{\sqrt{2k+1}+\sqrt{2k+2}}<\frac{1}{\sqrt{2k+1}+\sqrt{2k+2}}+\frac{1}{\sqrt{2k}+\sqrt{2k+1}}=\sqrt{2k+2}-\sqrt{2k}$
Cho $k=1;2;3;...;n-1;n$ rồi cộng 2 vế các BĐT lại ta có ĐPCM



#311448 Viết phương trình tiếp tuyến (P): $y=x^2$ //AB biết $A,B...

Posted by minhtuyb on 19-04-2012 - 13:49 in Các bài toán Đại số khác

Chú đừng khinh anh !!!
___________________________________
Spam nhiều quá rùi, thôi xóa đi đây

Cố spam cho các mod THPT xóa một thể ^_^:
Ơ đây là dạng bài lớp 9 mà, chỗ mình làm chán òi ="='
P/s: Chị cậu lớp mấy :)